Exam 2 Chapters 8,9,10, 19,24,27,32 prep U and TB

Ace your homework & exams now with Quizwiz!

Which statement made by a client who was recently admitted to the medical unit with a diagnosis of pneumonia indicates a physical inability to learn? "May I have something to eat?" "The pain in my chest has gone." "I am having difficulty breathing." "Finally, I am getting medical attention."

"I am having difficulty breathing.

Which statement made by a client who was recently admitted to the medical unit with a diagnosis of pneumonia indicates a physical inability to learn? "May I have something to eat?" "The pain in my chest has gone." "I am having difficulty breathing." "Finally, I am getting medical attention."

"I am having difficulty breathing."

A nurse is requesting to receive the change-of-shift report at the bedside of each client. The nurse giving the report asks about the purpose of giving it at the bedside. Which response by the nurse receiving the report is most appropriate? "It will allow for us to see the client and possibly increase client participation in care." "It will let me see everything that has been done and things that need to be done." "It makes our client feel like we care, especially if we start the day off with a clean room." "It will give me a better sense of what my workload will be today."

"It will allow for us to see the client and possibly increase client participation in care."

The nurse is caring for a client whose spouse wishes to see the electronic health record. What is the appropriate nursing response? "Let me get that for you." "Only authorized persons are allowed to access client records." "The provider will need to give permission for you to review." "I am sorry I can't access that information."

"Only authorized persons are allowed to access client records."

The nurse has provided education to a client about home care for an open surgical wound on the lower left extremity. When evaluating learning through the cognitive domain, what statement by the nurse would be appropriate? "Tell me about what signs of infection you will report to the health care provider." "I would like you to demonstrate how to change the dressing on your leg." "Let's see how you irrigate the wound with saline." "I notice that you do not have the dressing secured. Place a piece of tape on the wrap."

"Tell me about what signs of infection you will report to the health care provider.

1. Paramedics arrive in the emergency department with a client who was in a motor vehicle collision. The paramedic reports that the driver was restrained, the car was traveling about 30 miles per hour (48 km/hr), and the air bags were not deployed. The paramedic continues to report that the car was struck from behind and that all individuals in the car were able to self-extricate. Which statement made by the nurse is verifying the report from the paramedic?

- All of the people got themselves out of the car?

Assertive Nurse's Self-Presentation

- Confident; open body posture - Use of clear, concise "I" statements - Ability to share effectively one's thoughts, feelings, and emotions - Working to capacity with or without supervision - Remaining calm under supervision - Asking for help when necessary - Giving and accepting compliments - Admitting mistakes and taking responsibility for them

1. What are the four levels of communication?

- Intrapersonal (Self-talk; communication within a person) - Interpersonal (Occurs between two or more people with a goal to exchange messages) - Group (Small-group, Organizational communication)

Aggressive Behavior:

- Involves asserting one's rights in a negative manner that violates the rights of others - Can be verbal or physical - Communication is marked by tension and anger, - inhibiting the formation of good relationships and collaboration. - Characteristics include using an angry tone of voice, - making accusations, and demonstrating belligerence and intolerance. Focus is usually "winning at all costs."

Rapport builders

- Specific objectives - Comfortable environment - Privacy - Confidentiality - Patient vs. task focus - Utilization of nursing observations - Optimal pacing

1. What are the five Communication Process (Berlo)?

- Stimulus or referent - Sender or source of message (encoder) - Message itself - Medium or channel of communication - Receiver (decoder)

S B A R

-S: take 5 to 10 seconds to explain; situation for example "I am calling about", the patients code status is, "the problem im calling about" -B: provide context and data; background for example, "the patient mental status is" -A: describe specific problems/situation Assessment for example, this is what I think the problem is -R: explain what you want to do about it; recommendation for example, "from physician", "are any test needed, if a treatment is ordered then ask

Which is the proper way to document midnight in a client's record? 0000 2401 1200 1201

0000

The nurse is caring for an older adult client who states the need to use the restroom. Which safety intervention must the nurse perform first? Assess the need for assistance with ambulation. Put the client's bedside rails up. Apply socks to the client's feet. Arrange furniture so that the client has something to hold on to.

Assess the need for assistance with ambulation

1. A client is diagnosed with diabetes. The client's adult child offers to serve as an interpreter, because the client does not speak the dominant language. Which is the best action for the nurse to take?

Contact a professional interpreter

1. The nurse makes a contract with the client during which phase of the nurse-client relationship?

Correct response: Orientation phase

1. Nonverbal (body language)

Facial expressions, touch, eye contact o Posture, gait, gestures o General physical appearance o Mode of dress and grooming o Sounds, silence o Electronic communication

A nurse follows surgical asepsis techniques for inserting an indwelling urinary catheter in a client. What is an accurate guideline for using this technique? Hold sterile objects above waist level to prevent inadvertent contamination. Consider the outside of the sterile package to be sterile. Consider the outer 3-in. (8-cm) edge of a sterile field to be contaminated. Open sterile packages so that the first edge of the wrapper is directed toward the nurse.

Hold sterile objects above waist level to prevent inadvertent contamination.

After talking with a group of staff nurses on the medical-surgical unit, the nurse manager identifies a need for change. When developing a plan for change, which question would the nurse manager need to ask first? Is the behavior something that can be changed? Are there individuals within the group who might resist the change? Is the group ready for making the change? How quickly can the change be made?

Is the behavior something that can be changed?

A nurse leader on a unit allows the staff to make all decisions and direct themselves, including filling out the work schedule. The nurse leader is practicing which leadership style? Autocratic Democratic Laissez-faire Transformational

Laissez-faire

What is the most appropriate teaching strategy for the nurse to use for a 1-hour presentation on the prevention of osteoporosis to a group of 30 college-age women? Role play Lecture/discussion Demonstration Test taking

Lecture/discussion

In SBAR, what does R stand for? Reinforcing data Response Recommendations Report

Recommendations

A nurse manager best demonstrates effective leadership characteristic by which action? Being very structured and rigid with the unit flow Knowing all information about the unit processes Indicating an interest in becoming a role model Sharing a vision for the unit and enlisting support

Sharing a vision for the unit and enlisting support

The case manager works in a facility that supports a participative leadership style. Which action should the nurse recognize as an example of democratic leadership style? The nurse implementing the physician's written orders for client care The unlicensed assistive personnel (UAP) taking vital signs for several nurses The dietitian completing a nutritional assessment for a registered nurse (RN) The interdisciplinary health care team collectively developing plans of care for clients

The interdisciplinary health care team collectively developing plans of care for clients

A 56-year-old client meets with the nurse for education about a recently diagnosed atrial fibrillation. The client verbalizes concerns about being away from work too long and doubts about the necessity of having blood tests every week, as the client has no symptoms. Which is the best motivational statement by the nurse for this client? "Your doctor wants you to take your warfarin every day, go to the clinic every week to have blood drawn, and then wait for any dosage change. Do you understand?" "You have to take your warfarin and go to the clinic every week for a blood draw. It's not the most convenient way to live, but you have to do it." "The medicine and blood work can help prevent blood clots, which can lead to strokes. What do you know about warfarin therapy?" "Atrial fibrillation is when your upper heart beats ineffectively and blood clots can go to your brain. Would you like some printed information about this?"

The medicine and blood work can help prevent blood clots, which can lead to strokes. What do you know about warfarin therapy?"

The nurse is teaching a new nurse about preparing a sterile field. Which action made by the new nurse would indicate further teaching is required? The new nurse touches 1.5 in (4 cm) from the outer edges. The sterile field is set up at waist level. Direct visualization of the sterile field is maintained. The top flap of the package is opened away from the new nurse's body.

The new nurse touches 1.5 in (4 cm) from the outer edges

When a nurse is planning for learning, who must decide who should be included in the learning sessions? The health care team The doctor and nurse The nurse and the client The client and the client's family

The nurse and the client

When a nurse is planning for learning, who must decide who should be included in the learning sessions? The health care team The doctor and nurse The nurse and the client The client and the client's family

The nurse and the client

A nurse is caring for a client who is diagnosed with tuberculosis. Which nursing intervention promotes infection control based on nursing practice standards for safety? The nurse places the client in a private room with the door open. The nurse uses droplet precautions when providing care for the client. The nurse keeps visitors 3 feet away from the infected person. The nurse places the client in a private room with monitored negative air pressure.

The nurse places the client in a private room with monitored negative air pressure

The registered nurse (RN) wants to delegate a task to an unlicensed assistive personnel (UAP) but is unsure if the task can be delegated. What is the best resource for the RN to use in determining if the task is appropriate? The medical practice act website A nurse mentor or a nurse preceptor A more experienced nurse working on the same unit The nurse practice act or facility's policy and procedures

The nurse practice act or facility's policy and procedures

Termination

The patient will participate in identifying the goals accomplished or the progress made toward goals.

The registered nurse (RN) wants to delegate measuring a client's urinary output to an unlicensed assistive personnel (UAP). Which factors should the nurse consider before delegating the task? The complexity of the activity, age of the UAP, and predictability of the outcome Predictability of the UAP, the amount of time required for the task, and RN's skill level The stability of the client's condition, potential for harm, and complexity of the activity The context of the other client needs, the desired outcome, and autonomy of the client

The stability of the client's condition, potential for harm, and complexity of the activity

1. A dialysis nurse is educating a client on caring for the dialysis access that was inserted into the client's right arm. The nurse assesses the client's fears and concerns related to dialysis, the dialysis access, and care of the access. This information is taught over several sessions during the course of the client's hospitalization. Which phase of the working relationship is best described in this scenario?

The working phase

1. A client with a cardiac dysrhythmia was recently prescribed metoprolol and is at a follow-up appointment at the cardiologist's office. The client tells the nurse, "I feel depressed, tired, and I have no desire to exercise." To determine a cause-and-effect relationship, the nurse should ask:

Were you tired and depressed before starting the new medication?"

The nurse calls the health care provider due to changes in the client's status. Using the SBAR, the nurse is about to address Recommendation. Which statement appropriately supports this part of the SBAR? "I am concerned that the client might be exhibiting sepsis." "The client's temperature has been 102°F (38.9°C) for the last 6 hours." "The client was admitted today with a urinary tract infection." "Will you prescribe a complete blood count to check the white blood cell count and a culture?"

Will you prescribe a complete blood count to check the white blood cell count and a culture?

1. A nurse is completing a health history with a client being admitted for a mastectomy. During the interview the client states, "I do not know what to do. I am not sure if I really need this surgery." Which response by the nurse demonstrates active listening?

You seem unsure. Tell me your concerns about your surgery.

1. A nurse is assessing vital signs on a pregnant client during a routine prenatal visit. The client states, "I know labor will be so painful, it sounds awful. I am sure I will not be able to stand the pain; I really dread going into labor." What is the best response from the nurse?

You're worried about how you will tolerate the pain associated with labor."

A nurse is arranging for home care for clients and reviews the Medicare reimbursement requirements. Which client meets one of these requirements? a client who is homebound and needs skilled nursing care a client whose rehabilitation potential is not good a client whose status is stabilized a client who is not making progress in expected outcomes of care

a client who is homebound and needs skilled nursing care

A nursing faculty member is teaching a class of second-degree students who have an average age of 32. What is important to remember when teaching adult learners? A) a focus on the immediate application of new material B) a need for support to reduce anxiety about new learning C) older students may feel inferior in terms of new learning D) all students, regardless of age, learn the same

a focus on the immediate application of new material

The nurse is caring for an older adult with pulmonary tuberculosis. Which precautions will the nurse begin? airborne droplet contact none

airborne

The nurse is caring for a client with tuberculosis. The prior shift's nurse has placed the client in droplet precautions. Which is the appropriate nursing action? change to contact precautions change to airborne precautions change to standard precautions continue with droplet precautions

change to airborne precautions

A senior student nurse has been elected class president. What type of power will the student have in this position? A) explicit power B) assumed power C) absolute power D) implied power

explicit power

When charting the assessment of a client, the nurse writes, "Client is depressed." This documentation is an example of: factual statement. interpretation of data. important information. relevant data.

interpretation of data.

When caring for a psychiatric client, a nurse would make a formal contract with the client during which phase of the nurse-client relationship?

orientation phase

The nurse is providing care to a client with Lyme disease. The nurse identifies the vector of this infection as: fungus. parasite. virus. bacteria.

parasite

What is the primary focus of communication during the nursepatient relationship? A) time available to the nurse B) nursing activity to be performed C) patient and patient needs D) environment of the patient

patient and patient needs

A nurse is working with a 55-year-old woman diagnosed with human immunodeficiency virus (HIV). This nurse has another client that day who has an upper respiratory infection. What is the most important thing the nurse can do to prevent the client with HIV from acquiring the upper respiratory infection? wear gloves when touching the client wear a mask and gown in the client's room avoid direct contact with the client perform hand hygiene before and after entering the client's room

perform hand hygiene before and after entering the client's room

A nurse manager reviews an employee's contribution to the nursing division annually. This process is: interpreting quality indicators. employee's job satisfaction survey. performance appraisal. reward and development survey.

performance appraisal

A parent of a 9-year-old child states to the nurse, "I have not noticed any fever yet but my child describes feeling achy and not well." Which phase of the fever does the nurse identify the child may be experiencing? prodromal invasion stationary resolution

prodromal

A nurse is maintaining a problem-oriented medical record for a client. Which component of the record describes the client's responses to what has been done and revisions to the initial plan? data base problem list plan of care progress notes

progress notes

Which principle should guide the nurse's documentation of entries on the client's health care record? Correcting fluid is used rather than erasing errors. Documentation does not include photographs. Precise measurements should be used rather than approximations. Nurses should not refer to the names of physicians.

recise measurements should be used rather than approximations.

The most common infection in children is: respiratory. gastrointestinal. neurologic. urinary.

respiratory

The nurse is preparing a SOAP note. Which assessment findings are consistent with objective client data? pain rating of 4 on a scale of 0-10 describes wound as itchy urine output 100 ml concerned with feeling tired

urine output 100 ml

Which is not appropriate regarding the use of gowns as PPE? use of paper or cloth gowns donning a gown when splashing use of one gown per person per shift use of a new gown each time the nurse enters the room

use of one gown per person per shift

A nurse tells a patient, Why wont you get out of bed? Are you always this lazy? This is an example of which of the following barriers to communication? A) using comments that give advice B) using judgmental language C) using leading questions D) using probing questions

using judgmental language

1. A patient states I have been expericing complications of diabetes. The nurse needs to direct the patient to gain more information what is the most appropriate comment or question to elict additional information? A.do you take two injections of insulin to decrease the complication B. most HCP recommend diet and exercise to regulate blood sugar C. most complications of diabetes are related to neuropathy D. what specific complications have you experienced

what specific complications have you experienced

A client is scheduled for a CABG procedure. What information should the nurse provide to the client? "A coronary artery bypass graft will benefit your heart." "The CABG procedure will help identify nutritional needs." "A complete ablation of the biliary growth will decrease liver inflammation." "The CABG procedure will help increase intestinal motility and prevent constipation.

"A coronary artery bypass graft will benefit your heart

An older adult client tells the nurse, "I do not understand why I have had so many episodes of infection lately." How should the nurse respond? "It is possible that you are not washing your hands well enough." "As we age, our immune system does not function as well." "You will have to limit who comes to visit since they may be exposing you." "There are a lot of infectious processes around and there is nothing that can be done."

"As we age, our immune system does not function as well."

he parents of a hospitalized 10-year-old ask the nurse if they can review the health care records of their child. What is the appropriate response from the nurse? "I will arrange access for you to review the record after you put your request in writing." "No, the physician will not give you access to review the records." "Are you questioning the care of your child?" "Only the client has the right to review the health care records."

"I will arrange access for you to review the record after you put your request in writing.

The nurse is visiting a client who was released from inpatient rehabilitation 6 weeks ago after a 5-month recovery from a motor vehicle accident that left the client immobile. As the nurse enters the home, the client braces hands on the arms of a chair to rise and uses crutches to walk across the room. What is the best response by the nurse? "Let me document that you can walk." "Those physical therapists work wonders. "You have made an amazing recovery." "Are you supposed to be out of the wheelchair?"

"You have made an amazing recovery."

What part of the patients record is commonly used to document specific patient variables, such as vital signs? A) progress notes B) nursing notes C) critical paths D) graphic record

D) graphic record

A pregnant client presents to the emergency department with vaginal bleeding. A transvaginal ultrasound is performed, and the health care provider informs the client that there are normal fetal heart tones noted. The client begins to tear-up and has a worried appearance. To facilitate therapeutic communication, what statement would the nurse make after observing the client's nonverbal communication?

- Take your time and tell me how you are feeling. I have plenty of time to answer your questions and discuss any thoughts or feelings with you."

1. A nurse has been caring for a client who had a myocardial infarction 2 days ago. During the morning assessment, the nurse asks the client how the client feels. Which scenario warrants further investigation?

- The client stares at the floor and states, "I feel fine."

Working phase:

- The patient will actively participate in the relationship. - The patient will cooperate in activities that work toward achieving mutually acceptable goals. - The patient will express feelings and concerns to the nurse.

6. Orientation phase:

- The patient will call the nurse by name - The patient will accurately describe the roles of the participants in the relationship - The patient and nurse will establish an agreement about: o Goals of the relationship o Location, frequency, and length of the contacts o Duration of the relationship.

Dispositional traits

- Warmth and friendliness - Openness and respect - Empathy - Honesty, authenticity, trust - Caring - Competence

1. A nurse suspects that a client may have a hearing problem. The nurse should attempt to consult:

- an audiologist.

A client is being screened for a parasitic infection and the physician orders stool specimens. When explaining to the client about collecting the specimens, the nurse would inform the client that the specimens will be collected daily for: 2 days. 3 days. 4 days. 5 days.

3 days

A nurse is counseling several clients for depression. Four of them do not seem to be improving, which leads the nurse to suggest a referral to a psychiatric nurse practitioner. Which of these clients would be most likely to attend the scheduled appointment? A 45-year-old female who is unsure of the benefit of psychiatric care, on a fixed income, and has good family support A 51-year-old male who walks to most places because of a lack of transportation, has a low income, and works days A 36-year-old male who uses public transportation, is unable to read, and wants to confer with a pastor A 28-year-old female who works nights, is willing to try, and asks about insurance coverage of the appointment

A 28-year-old female who works nights, is willing to try, and asks about insurance coverage of the appointment

Which statement describes the person who is likely the most motivated to learn? A 29-year-old male whose significant other is insisting on the client receiving the education A 52-year-old male who has been hired to drive the client home from the clinic A 70-year-old female who is the client's spouse and is learning the care so the client can come home A 25-year-old female who just completed a course of physical therapy

A 70-year-old female who is the client's spouse and is learning the care so the client can come home

which clinical situation is addressed by the provisions of the Health Insurance Portability and Accountability Act (HIPAA)? A client has asked a nurse if he can read the documentation that his physician wrote in his chart. A client wishes to appeal her insurance company's refusal to reimburse for a diagnostic test. A client has asked for a second opinion regarding treatment options for her diagnosis of ovarian cancer. A client who resides in Indiana has required hospitalization during a vacation in Hawaii.

A client has asked a nurse if he can read the documentation that his physician wrote in his chart

A nurse strives to establish trusting interpersonal relationships with patients, peers, subordinates, and superiors to facilitate goal achievement and personal growth of all participants. Which of the following skills is this nurse demonstrating? A) communication skills B) problem-solving skills C) management skills D) self-evaluation skills

A) communication skills

A nurse is preparing an operating room theatre for a surgical procedure. Which point regarding the principles of surgical asepsis should the nurse keep in mind when preparing sterilized surgical instruments? When a sterile item touches something that is not sterile, it may not be contaminated. Any partially uncovered sterile package need not be considered contaminated. A commercially packaged surgical item is not considered sterile if past expiration date. Sterility may not be preserved even when one sterile item touches another sterile item.

A commercially packaged surgical item is not considered sterile if past expiration date.

The nurse is conducting an in-serve on change theories. Which example should the nurse include in the presentation illustrating Lewin's three stages of change? Nurses work in groups to care for clients. Physicians collaborate with nurses. Quality client care is provided. Staff voice concerns among themselves about the requirement of mandatory overtime. A unit meeting is called to discuss the issues with management. The mandatory overtime requirement is reinforced by management. A need for a safer intravenous therapy pump is identified. New intravenous therapy pumps are purchased and training for staff nurses is provided. Intravenous therapy pumps are placed on the nursing unit for client care. Multifunctional machines monitor clients' vital signs. Data from the machines is used to create trends in clients' health. The trends are compared to national norms and used to determine if measures can be taken to improve client care.

A need for a safer intravenous therapy pump is identified. New intravenous therapy pumps are purchased and training for staff nurses is provided. Intravenous therapy pumps are placed on the nursing unit for client care.

What nursing care behavior by the nurse engenders a client's trust in the nurse?

A nurse answers the client's questions about an upcoming test in a calm gentle voice while making eye contact with the client.

Which nurse would most likely be the best communicator?

A nurse who easily develops a rapport with clients

A nurse is following a clinical pathway that guides the care of a client after knee surgery. When the nurse observes the client vomiting, it creates a deviation from the clinical pathway. What should the nurse identify this event as? A never event A variance An audit A sentinel event

A variance

How long should a healthcare worker scrub hands that are not visibly soiled for effective hand hygiene? A) 15 seconds B) 30 seconds C) 1 minute D) 5 minutes

A) 15 seconds

A nurse is described as a quantum leader. Which of the following actions characterize this type of leadership? A) A nurse conducts a blind survey to evaluate her leadership skills. B) A nurse relinquishes power to a group deciding hospital policy. C) A nurse makes policy decisions for coworkers without consulting them. D) A nurse sticks to the tried and true methods when implementing patient care.

A) A nurse conducts a blind survey to evaluate her leadership skills.

A patient with an upper respiratory infection (common cold) tells the nurse, I am so angry with the nurse practitioner because he would not give me any antibiotics. What would be the most accurate response by the nurse? A) Antibiotics have no effect on viruses. B) Let me talk to him and see what we can do. C) Why do you think you need an antibiotic? D) I know what you mean; you need an antibiotic.

A) Antibiotics have no effect on viruses

Which of the following strategies might a nurse use to increase compliance with teaching? A) Include the patient and family as partners. B) Use short, simple sentences for all ages. C) Provide verbal instruction at all times. D) Maintain clear role as the authority.

A) Include the patient and family as partners.

The ANA, which is committed to monitoring the regulation, education, and use of NAPs, recommends adherence to which one of the following principles? A) It is the nursing profession that determines the scope of nursing practice. B) It is the RN who defines and supervises the education, training, and use of any unlicensed assistant roles. C) It is the assigned NAP who is responsible and accountable for his or her nursing practice. D) It is the purpose of the RN to work in a supportive role to the assistive personnel

A) It is the nursing profession that determines the scope of nursing practice.

A nurse is using motivational interviewing to find out why a patient refuses to participate in the recommended rehabilitation program. Which of the following is an example of using the skill of reflective listening to help motivate this patient? A) So, you feel that you are not ready to start a program this week? B) Why do you feel that you are not ready to start rehabilitation? C) I understand that you are afraid to start rehabilitation; where do you see yourself in a week? D) Remember we discussed what needs to be done to get you back on your feetHow do you feel about getting started?

A) So, you feel that you are not ready to start a program this week?

. A nurse is teaching a home care patient how to administer a topical medication. The patient is watching television while the nurse is talking. What might be the result of this interaction? A) A) The message will likely be misunderstood. B) The stimulus for communication is unclear. C) The receiver will accurately interpret the message. D) The communication will be reciprocal.

A) The message will likely be misunderstood.

A nurse is positioning a sterile drape to extend the working area when performing a urinary catheterization. Which of the following is an appropriate technique for this procedure? A) Use sterile gloves to handle the entire drape surface. B) Fold the lower edges of the drape over the sterile-gloved hands. C) Touch only the outer 2 inches of the drape when not wearing sterile gloves. D) When reaching over the drape do not allow clothing to touch the drape.

A) Use sterile gloves to handle the entire drape surface.

A woman tests positive for the human immunodeficiency virus antibody but has no symptoms. She is considered a carrier. What component of the infection cycle does the woman illustrate? A) a reservoir B) an infectious agent C) a portal of exit D) a portal of entry

A) a reservoir

A nurse has seen several patients at a community health center. Which of the patients would be most at risk for developing an infection? A) an older adult with several chronic illnesses B) an infant who has just received first immunizations C) an adolescent who had a basketball physical D) a middle-aged adult with joint pain and stiffness

A) an older adult with several chronic illnesses

What is the primary purpose of the patient record? A) communication B) advocacy C) research D) education

A) communication

A nurse uses informatics to plan nursing care for a patient. Which three terms best describes this science as it is applied to nursing? A) data, information, knowledge B) process, documentation, analysis C) research, controls, variables D) hypothesis, nursing, practice

A) data, information, knowledge

What type of leader shares decisions and activities with group participants? A) democratic B) autocratic C) laissez-faire D) situational

A) democratic

A nurse is caring for an adolescent who is diagnosed with mononucleosis, commonly called the kissing disease. The nurse explains that the organisms causing this disease were transmitted by: A) direct contact. B) indirect contact. C) airborne route. D) vectors.

A) direct contact.

What is the primary purpose of an incident report? A) means of identifying risks B) basis for staff evaluation C) basis for disciplinary action D) format for audiotaped report

A) means of identifying risks

Mrs. Treem is on isolation because she acquired a methicillin-resistant S. aureus (MRSA) infection after hospitalization for hip replacement surgery. What name is given to this type of infection? A) nosocomial B) viral C) iatrogenic D) antimicrobial

A) nosocomial

Andrew has just graduated with a baccalaureate degree in nursing. What type of nursing leadership will he be expected to provide? A) nursing care of the individual patient B) demonstration of selected critical skills C) ability to be a follower rather than a leader D) nursing care of groups of patients

A) nursing care of the individual patient

A nurse organizes patient data using the SOAP format. Which of the following would be recorded under S of this acronym? A) patient complaints of pain B) patient symptoms C) patients chief complaint D) patient interventions

A) patient complaints of pain

Which one of the following methods of documentation is organized around patient diagnoses rather than around patient information? A) problem-oriented medical record (POMR) B) source-oriented record C) PIE charting system D) focus charting

A) problem-oriented medical record (POMR)

A nurse working in a physicians office uses the managerial function known as organizing. What is involved in this function? A) resources B) problems C) workforce D) evaluation

A) resources

Which of the nursing roles is primarily performed during the working phase of the helping relationship? A) teacher and counselor B) provider of care C) leader and manager D) researcher

A) teacher and counselor

A diabetes nurse educator is teaching a patient, newly diagnosed with diabetes, about his disease process, diet, exercise, and medications. What is the goal of this teaching? A) to help the patient develop self-care abilities B) to ensure the patient will return for follow-up care C) to facilitate complete recovery from the disease D) to implement ordered teaching and counseling

A) to help the patient develop self-care abilities

A nurse is performing a sterile dressing change. If new sterile items or supplies are needed, how can they be added to the sterile field? A) with sterile forceps or hands wearing sterile gloves B) by carefully handling them with clean hands C) with clean forceps that touch only the outermost part of the item D) by clean hands wearing clean latex gloves

A) with sterile forceps or hands wearing sterile gloves

A student has reviewed a patients chart before beginning assigned care. Which of the following actions violates patient confidentiality? A) writing the patients name on the student care plan B) providing the instructor with plans for care C) discussing the medications with a unit nurse D) providing information to the physician about laboratory data

A) writing the patients name on the student care plan

The nurse is teaching an 80-year-old client how to instill eye drops for glaucoma. The client's daughter asks, "How do you know that my mother understands what to do?" What is the appropriate nursing response? A. "After I demonstrate it once, your mother will be able to do it." B"When 15 minutes have passed, I will ask your mother to show me how to instill the drops." C. "We can never be completely sure that your mother understands instructions." D. "I will have you bring your mother back next week to see how things are going."

A. "After I demonstrate it once, your mother will be able to do it."

1. When interacting with a patient the nurse answers I am sure everything will be fine you have nothing to worry about this is an example of what type of inappropriate communication technique? A. Cliché B. Giving advice C. Being judgmental D. Changing the subject

A. Cliché

1. A nurse is caring for a pt who is hospitalized following a double mastectomy is preparing for discharge plan for the patient. Which action should be the focus of this termination phase of the helping relationship? A. Determining the progress made in achieving established goals B. Clarifying when the patient should take medications C. Reporting the progress made in teaching to the staff D. Including all family members in the teaching session

A. Determining the progress made in achieving established goals Rationale:the termination phase occurs when the conculsionof the initial agreement is acknowledged. Discharge planning coordinates with the termination phase of a helping relationship the nurse should determine the progress made in achieving the goals related to patient care

A public health nurse is leaving the home of a young mother who has a special needs baby the neighbor states "how is she doing, since the baby father is no help?" what is the best response to the neighbor?

A. New mothers need support

Why is communication important to the assessing step of the nursing process? A) The major focus of assessing is to gather information. B) Assessing is primarily focused on physical findings. C) Assessing involves only nonverbal cues. D) Written information is rarely used in assessment.

A. The major focus of assessing is to gather information.

1. A nurse notices a patient is walking to the bathroom with a stooped gait, facial grimacing, and gasping sounds based on these nonverbal clues, for which condition would the nurse assess? A. Pain B. Anxiety C. Depression D. Fluid volume deficit

A.pain

The new nurse is having difficulty managing the time required to care for a group of complex clients and is several hours behind in completing nursing interventions. Which intervention should the nurse complete first? Administer a dose of digoxin that is two hours behind schedule. Perform a dressing change to an abdominal abscess that is three hours behind schedule. Obtain discharge orders for a client who is ready to be transferred to a long-term nursing facility. Complete a medication reconciliation form on a client who has recently been admitted to the hospital.

Administer a dose of digoxin that is two hours behind schedule

Which guideline is most important for the nurse to keep in mind when planning to teach an exercise class to a group of older adults? Allow ample time for psychomotor skills. Keep the session at 2 to 3 hours. Allow for long-term memory loss. Provide information in a structured format.

Allow ample time for psychomotor skills.

Which term describes foreign particles that enter a host and stimulate the body's immune response? Macrophage Phagocyte Antibody Antigen

Antigen

The nurse is teaching a client with diabetes how to inject daily insulin. Which method is most effective in evaluating the teaching? Ask the client to repeat the steps of injection in order. The nurse uses a brochure to explain how to give an injection. Ask the client to demonstrate how to self-inject the morning insulin. Provide a teaching session that includes a question and answer discussion.

Ask the client to demonstrate how to self-inject the morning insulin

When preparing client teaching materials, how does the nurse best assess a client's preferred learning style? Observe the client's behaviors. Provide teaching that works for the broadest base of clients. Ask the client, "Do you learn best by observing, valuing, or doing?" Determine client learning needs based on age and ability to hear effectively.

Ask the client, "Do you learn best by observing, valuing, or doing?"

A nurse is working with a 15-year-old client with sickle cell anemia. The client was started on a new pain management plan today, and the nurse is evaluating the effectiveness of the plan. Which is not appropriate to include in the nursing care? Asking only the client's parents to be present at the education session Including a note about who was taught this new information in the client's chart Assuring the client that the conversation is confidential except under extreme circumstances Answering questions openly and honestly

Asking only the client's parents to be present at the education session

When caring for a client at the health care facility, the nurse observes that the client is having difficulty understanding the health education. Which action is most appropriate? Assess for cultural differences. Boost the morale of the client. Delegate the health education to a colleague. Replace one-on-one teaching with written materials.

Assess for cultural differences

When caring for a client at the health care facility, the nurse observes that the client is having difficulty understanding the health education. Which action is most appropriate? Assess for cultural differences. Boost the morale of the client. Delegate the health education to a colleague. Replace one-on-one teaching with written materials.

Assess for cultural differences.

1. A nurse is completing a health history on a client who has a hearing impairment. Which action should the nurse take first to enhance communication?

Assess how the client would like to communicate

A client informs the nurse about being committed to quitting smoking to improve health. During discussion, the nurse asks the client "on a scale of 0 to 10, how likely are you to attend a support group?" Which strategy of motivational interviewing is the nurse using with the client? Assessing importance Elicit-provide-elicit Evoking change talk Prioritizing

Assessing importance

A nurse manager informs the staff members during a meeting that unlicensed assistive personnel will no longer be allowed to check clients' blood glucose levels. The nurse manager informs the group that this is a new policy on the unit and that discussions will not change the enforcement of this policy. What type of leadership style is the nurse manager demonstrating? Autocratic Democratic Laissez-faire Transformational

Autocratic

A client is received into the emergency department after getting shot in the chest. The client is hemorrhaging profusely and is in hypovolemic shock. The nurse calls a code blue. What type of leadership style will be most effective during the management of the code? Autocratic leadership Laissez-faire leadership Democratic leadership Transactional leadership

Autocratic leadership

Planned change is a purposeful, systematic effort to alter or bring about change. What occurs next after alternative solutions to a problem are determined and analyzed? A) All of the alternative solutions are implemented. B) A course of action is chosen from among the alternatives. C) The effects of the change are evaluated. D) The change is stabilized and established

B) A course of action is chosen from among the alternatives

Alice Jones, a registered nurse, is documenting assessments at the beginning of her shift. How should she sign the entry? A) Alice J, RN B) A. Jones, RN C) Alice Jones D) AJRN

B) A. Jones, RN

Which of the following statements is true of healthcare personnel and good hand hygiene? A) Hand hygiene is carefully followed. B) Compliance is difficult to achieve. C) Only nurses need to practice hand hygiene. D) Wearing gloves reduces the need for hand hygiene.

B) Compliance is difficult to achieve.

A nurse is teaching adolescents how to prevent infections. What statement by one of the adolescents indicates that more teaching is needed? A) I will wash my hands before and after going to the bathroom. B) I dont wear a condom when I have sex, but I know my partners. C) I always eat fruits and vegetables, and I sleep 8 hours a night. D) When I have an infection, I rest and take my medications.

B) I dont wear a condom when I have sex, but I know my partners

A nurse refers a patient with a new colostomy to a support group. This nurse is practicing which of the following aims of nursing? A) promoting health B) preventing illness C) restoring health D) facilitating coping

D) facilitating coping

When comparing team nursing with functional nursing, what characteristic is found? A) Team nursing is very similar to functional nursing. B) Team nursing focuses on individual patient care. C) Functional nursing has a stronger focus on the patient. D) Functional nursing is based on total patient care

B) Team nursing focuses on individual patient care.

A nurse is sitting near a patient while conducting a health history. The patient keeps edging away from the nurse. What might this mean in terms of personal space? A) The nurse is too far away from the patient. B) The nurse is in the patients personal space. C) The patient does not like the nurse. D) The patient has concerns about the questions.

B) The nurse is in the patients personal space.

A patient who has had abdominal surgery develops an infection in the wound while still hospitalized. Which of the following agents is most likely the cause of the infection? A) virus B) bacteria C) fungi D) spores

B) bacteria

Which of the following methods of documenting patient data is least likely to hold up in court if a case of negligence is brought against a nurse? A) problem-oriented medical record B) charting by exception C) PIE charting system D) focus charting

B) charting by exception

A nurse asks a patient to tell him the side effects of a medication. What learning domain is the nurse evaluating? A) affective B) cognitive C) psychomotor D) emotional

B) cognitive

How is change viewed in the new quantum age currently experienced in society? A) constant and predictable B) dynamic and constantly unfolding C) evolving very slowly D) an entity needing planning

B) dynamic and constantly unfolding

Developing a teaching plan is comparable to what other nursing activity? A) documenting in the nurses notes B) formulating a nursing care plan C) performing a complex technical skill D) using a standardized form or format

B) formulating a nursing care plan

A nursing home recently has had a significant number of nosocomial infections. Which of the following measures might be instituted to decrease this trend? A) mandating antibiotics for all nursing home residents B) having written infection-prevention practices for all employees C) requiring all employees to have monthly screenings for skin flora D) restricting visitors and community activities for residents

B) having written infection-prevention practices for all employees

When is the best time to evaluate ones own teaching effectiveness? A) during the teaching session B) immediately after a teaching session C) 1 week after the teaching session D) 1 month after the teaching session

B) immediately after a teaching session

A physicians order reads up ad lib. What does this mean in terms of patient activity? A) may walk twice a day B) may be up as desired C) may only go to the bathroom D) must remain on bed rest

B) may be up as desired

A nurse is documenting information about a patient in a long-term care facility. What is used in a Medicare-certified facility as a comprehensive assessment and as the foundation for the Resident Assessment Instrument (RAI)? A) PIE system B) minimum data set C) OASIS D) charting by exception

B) minimum data set

A nurse in a neighborhood clinic is conducting educational sessions on weight loss. What aim of nursing is met by these educational programs? A) practicing advocacy B) preventing illness C) restoring health D) facilitating coping

B) preventing illness

A college-aged student has influenza. At what stage of the infection is the student most infectious? A) incubation period B) prodromal stage C) full stage of illness D) convalescent period

B) prodromal stage

In what type of documentation method would a nurse document narrative notes in a nursing section? A) problem-oriented medical record B) source-oriented record C) PIE charting system D) focus chartin

B) source-oriented record

Which of the following types of skills is not needed for nursing leadership? A) communication skills B) technical skills C) problem-solving skills D) self-evaluation skills

B) technical skills

1. A nurse in the rehabilitation division states to the head nurse I need the day off and you didn't give it to me the head nurse replies well I wasn't aware you needed the day off and it isn't possible since staffing is so inadequate instead of this exchange what communication by the nurse would have been more effective? A. I placed a request to have 8th of august off, but im working and I have a doctors appointment B. I would like to discuss my scheduled with you I requested the 8th of august off the doctors appointment. Could I make a appointment C. I will need to call in on the 8th of august off, because I have a doctors appointment D. Since you didn't give me the 8th of august off will I need to find someone to work for me

B. I would like to discuss my scheduled with you I requested the 8th of august off the doctors appointment. Could I make a appointment

1. A nurse is providing instructions to a patient instructions to a patient regarding the procedure to change a colostomy bag during the teaching session the patient asks what type of of foods should I avoid to prevent gas? The patients questions allows for what type of communication on the nurse part? A. A closed ended answer B. Information clarification C. The nurse to give advice D. Assertive behavior

B. Information clarification

1. A nursing is preparing to administer morning care to a patient. What is the most important question that the nursing student ask the patient about personal hygiene? A. Would you prefer bath or shower B. May I help you with a bed bath now or later this morning C. I will be giving you your bath do you use soap or shower gel D. I prefer a showing in the evening when would, when would you like your bath

B. May I help you with a bed bath now or later this morning

. The family of a patient in a burn unit asks the nurse for information. The nurse sits with the family and discusses their concerns. What type of communication is this? A) intrapersonal B) interpersonal C) organizational D) focused

B. interpersonal

. A nurse uses the SBAR method to hand off the communication to the healthcare team. Which of the following might be listed under the B of the acronym? A) vital signs B) mental status C) patient problem D) further testing

B. mental status

The registered nurse on a busy telemetry floor is delegating tasks to an unlicensed assistive person (UAP). Which task is appropriate for the UAP to complete? Bathing a combative client Inserting an indwelling urinary catheter Taking an order over the telephone from a physician Assessing a client's wound

Bathing a combative client

The nurse is caring for a 60-year-old client with an improper bowel movement regimen. Which is the most appropriate method for the nurse to use in teaching this client? Refer the client to internet resources on proper bowel health. Have the client join a small group of other clients with the same problem and facilitate group discussions. Begin the session with a reference to the client's actual experience. Talk to the client's relatives and get a detailed account of the client's history.

Begin the session with a reference to the client's actual experience

The nurse is working at a facility that is applying for Magnet® Recognition. The nurse knows that compared with other hospitals, Magnet® hospitals have which direct effect on client care? Higher nurse retention Longer client stays Better client outcomes Improved job satisfaction scores

Better client outcomes

A nurse is educating a 4-year-old client about cast care following a tibia-fibula fracture. Which action is not developmentally appropriate to include in the nurse's teaching? Blocking 30 minutes of time for skill teaching Using dolls to demonstrate psychomotor skills Ensuring the client's parents are present Giving stickers as a reward for task completion

Blocking 30 minutes of time for skill teaching

Which of the following is an example of nonverbal communication? A) A nurse says, I am going to help you walk now. B) A nurse presents information to a group of patients. C) A patients face is contorted with pain. D) A patient asks the nurse for a pain shot.

C) A patients face is contorted with pain.

A nurse has completed morning care for a patient. There is no visible soiling on her hands. What type of technique is recommended by the CDC for hand hygiene? A) Do not wash hands, apply clean gloves. B) Wash hands with soap and water. C) Clean hands with an alcohol-based handrub. D) Wash hands with soap and water, follow with handrub.

C) Clean hands with an alcohol-based handrub.

Which of the following statements is true of factors that influence communication? A) Nurses provide the same information to all patients, regardless of age. B) Men and women have similar communication styles. C) Culture and lifestyle influence the communication process. D) Distance from a patient has little effect on a nurses message.

C) Culture and lifestyle influence the communication process.

A nurse manager has encountered resistance to a planned change. What is one way the nurse can overcome the resistance? A) Tell the staff that if they dont like it, they can quit. B) Implement change rapidly and all at once. C) Encourage open communication and feedback. D) Let the staff know that the change is mandated.

C) Encourage open communication and feedback.

A nurse is caring for a patient who is visually impaired. Which of the following is a recommended guideline for communication with this patient? A) Ease into the room without acknowledging presence until the patient can be touched. B) Speak in a louder tone of voice to make up for lack of visual cues. C) Explain reason for touching patient before doing so. D) Keep communication simple and concrete.

C) Explain reason for touching patient before doing so.

Which of the following data entries follows the recommended guidelines for documenting data? A) Patient is overwhelmed by the diagnosis of pancreatic cancer. B) Patient kidneys are producing sufficient amount of measured urine. C) Following oxygen administration, vital signs returned to baseline. D) Patient complained about the quality of the nursing care provided on previous shift.

C) Following oxygen administration, vital signs returned to baseline.

A patient asks to see his medical record (chart). How would the nurse respond? A) I cant let you do that without a doctors order. B) Our hospital policy is that you cant do that. C) I will get your chart and provide you with privacy to read it. D) Why would you want to do that? It will only make you worry.

C) I will get your chart and provide you with privacy to read it.

A nurse has access to computerized standardized plans of care. After printing one for a patient, what must be done next? A) Date it and put it in the patients record. B) Sign it and put it in the Kardex. C) Individualize it to the specific patient. D) Use it as printed, based on common needs.

C) Individualize it to the specific patient.

A patient comes to the emergency department with major burns over 40% of his body. Although all of the following are true, which one would provide the rationale for a nursing diagnosis of Risk for Infection? A) Stress may adversely affect normal defense mechanisms. B) White blood cells provide resistance to certain pathogens. C) Intact skin and mucous membranes protect against microbial invasion. D) Age, race, sex, and hereditary factors influence susceptibility to infection

C) Intact skin and mucous membranes protect against microbial invasion.

A nurse is considering the delegation of administering medications to an unskilled assistant. What is the first question the nurse must ask herself before doing so? A) Has the assistant been trained to perform the task? B) Have I evaluated the patients response to this task? C) Is the delegated task permitted by law? D) Is appropriate supervision available?

C) Is the delegated task permitted by law?

Which of the following is a characteristic of mentorship? A) It is a paid position to orient new nurses to the workplace. B) It involves membership in a professional organization. C) It is a link to a protg with common interests. D) It is not encouraged in healthcare settings

C) It is a link to a protg with common interests.

A young mother asks the nurse in a pediatric office for information about safety, diet, and immunizations for her baby. Which nursing diagnosis would be appropriate for this patient? A) Knowledge Deficit: Infant care B) Impaired Health Maintenance C) Readiness for Enhanced Parenting D) Readiness for Enhanced Copin

C) Readiness for Enhanced Parenting

A nurse is caring for a patient with a serious bacterial infection. The patient is dehydrated. Knowledge of the physical effects of the infection would support which of the following nursing diagnoses? A) High Risk for Infection B) Excess Fluid Volume C) Risk for Imbalanced Body Temperature D) Risk for Latex Allergy Response

C) Risk for Imbalanced Body Temperature

Which of the following statements accurately describes the use of power by change agents? A) They know that power comes from one sourcemanagement. B) When introducing change they do not enlist the support of key power players. C) They are often accomplished professional women. D) They do not recognize their own strengths and weaknesses.

C) They are often accomplished professional women.

Which of the following most accurately defines an infection? A) an illness resulting from living in an unclean environment B) the result of lack of knowledge about food preparation C) a disease resulting from pathogens in or on the body D) an acute or chronic illness resulting from traumatic injury

C) a disease resulting from pathogens in or on the body

In general, how do most people view change? A) by how it affects the cohesiveness of the group B) by how much it will cost in time and resources C) by how they are affected personally D) by how it will affect others on the staff

C) by how they are affected personally

Which of the following is an essential component of the definition of learning? A) increases self-esteem B) decreases stress C) can be measured D) cannot be measured

C) can be measured

A mother of a toddler wants to learn how to do CPR. What teaching strategy would be most effective in helping her learn? A) lecture B) discussion C) demonstration D) discovery

C) demonstration

A senior student has been elected president of the Student Nurses Association. Which of the following qualities is essential to being a nursing leader? A) physical stamina B) physical attractiveness C) flexibility D) independence

C) flexibility

Which of the following is an example of the bodys defense against infection? A) racial characteristics B) body shape and size C) immune response D) level of susceptibility

C) immune response

What patient characteristic is important to assess when using the health belief model as the framework for teaching? A) developmental level B) source of information C) motivation to learn D) family support

C) motivation to learn

What is the primary purpose of focus charting? A) nursing diagnoses B) medical problems C) patient concerns D) expected outcomes

C) patient concerns

What is the nurses best defense if a patient alleges nursing negligence? A) testimony of other nurses B) testimony of expert witnesses C) patients record D) patients' family

C) patients record

Of all possible nursing interventions to break the chain of infection, which is the most effective? A) administering medications B) providing good skin care C) practicing hand hygiene D) wearing gloves at all times

C) practicing hand hygiene

An elderly patient is very stressed about who will care for his pets while he is hospitalized for a fall that caused a fractured hip and hospitalization. What type of counseling would the nurse conduct? A) none B) long-term C) short-term D) motivational

C) short-term

A nurse caring for a patient who has gas gangrene knows that this infection originated in which of the following reservoirs? A) other people B) food C) soil D) animals

C) soil

The latest CDC guidelines designate standard precautions for all substances except which of the following? A) urine B) blood C) sweat D) vomitus

C) sweat

1. A nursing student is nervous and concerned about working at a clinical facility which action would best decrease anxiety and ensure success in the students provision of patient care? A. Determining the establish goals of the institution B. Ensuring that verbal and nonverbal communication is congruent C. Engaging in self talk to plan the day and decrease fear D. Speaking with fellow colleagues about how they feel

C. Engaging in self talk to plan the day and decrease fear

Which of the following is a characteristic of the helping relationship? A) it occurs spontaneously B) it is similar to a social relationship C) it is an unequal sharing of communication D) it is based on the needs of the nurse

C. it is an unequal sharing of communication

A nurse gives a speech on nutrition to a group of pregnant women. What is the speech itself known as? A) stimulus B) source C) message D) channel

C. message

What is the goal of the nurse in a helping relationship with a patient? A) to provide hands-on physical care B) to ensure safety while caring for the patient C) to assist the patient to identify and achieve goals D) to facilitate the patients interactions with others

C.to assist the patient to identify and achieve goals

A nurse was informed that a family member was involved in a car accident and transported to the emergency department in the same facility. What action by the nurse best demonstrates understanding of client privacy? Calling the client information desk to find out the room number of the family member Finding the emergency medical technicians who transported the family members and inquiring about the injuries Asking the emergency department nurse for information on the family member Accessing the electronic health record of the family member to find out extent of injury

Calling the client information desk to find out the room number of the family member

In the hospital setting, a nurse is responsible for overseeing the quality and financial outcomes of client care while working collegially with health care providers. What type of professional nursing practice is this considered? Functional nursing Case management Primary nursing Nurse mentorship

Case management

The nurse is assigned to change the client's abdominal dressing after hernia repair. Which action will the nurse perform? Change the dressing using aseptic technique. Change the dressing using clean gloves and sterile dressings. Change the dressing using sterile technique. Change the dressing using sterile gloves and clean dressings.

Change the dressing using sterile technique

Which documentation example best reflects the complexity of client teaching by the nurse? "Told client to take antibiotic as ordered." "Client return demonstrated how to use glucometer." "Taught client about peak flows; client verbalized understanding." "Client and spouse taught how to use phone app to count carbohydrates; client return demonstrated carb counting for a hypothetical meal."

Client and spouse taught how to use phone app to count carbohydrates; client return demonstrated carb counting for a hypothetical meal

A client reads the nutritional chart and follows it accurately. The nurse also notes that the client understands the need for a balanced diet and its relationship with a quick recovery. In which domain is the client demonstrating successful learning? Cognitive Affective Psychomotor Interpersonal

Cognitive

When caring for a client, the nurse observes that the client enjoys reading books and magazines. In which learning domain does the client's learning style fall? Cognitive Affective Psychomotor Interpersonal

Cognitive

The nurse has educated the client on the pathophysiology of osteoarthritis and degenerative joint disease. This type of teaching best illustrates which learning theory? Adaptive learning theory Behavioral learning theory Cognitive learning theory Developmental learning theory

Cognitive learning theory

A nurse manager is trying to resolve a conflict between the day and night shifts. The nurse manager wants to convince the involved persons to set aside their differences, determine a priority common goal having to do with improved client care, and accept mutual responsibility for achieving this goal. The nurse manager is using which type of conflict resolution? Avoiding Collaborating Competing Compromising

Collaborating

The nurse manager for the psychiatric unit sees that there are major conflicts between the day and night shift staff. The nurse manager suggests that each shift put aside their differences for a time and determine a common major goal. Which conflict resolution style does the nurse manager display? Avoiding Competing Collaborating Smoothing

Collaborating

Two staff nurses on a unit disagree with one another over certain key aspects of providing client care. The nurse manager of the unit arranges a meeting with the charge nurse and the two staff nurses at a mutually agreed-upon time to discuss this situation. This activity is most accurately described as what? Conflict resolution Problem solving Delegation Client protection

Conflict resolution

The nurse documents a progress note in the wrong client's electronic medical record (EMR). Which action would the nurse take once realizing the error? Immediately delete the incorrect documentation. Create an addendum with a correction. Contact information technology (IT) staff to make the correction. Contact the health care provider.

Create an addendum with a correction

he nurse documents a progress note in the wrong client's electronic medical record (EMR). Which action would the nurse take once realizing the error? Immediately delete the incorrect documentation. Create an addendum with a correction. Contact information technology (IT) staff to make the correction. Contact the health care provider.

Create an addendum with a correction.

The nurse is inserting an indwelling catheter. What steps will the nurse distinguish as priority in preventing an infection? Create an area for sterile field and opening packages Place water-soluble lubricant on catheter tip prior to insertion Wash the perineal area with soap and water Ensure opening port of the catheter is closed

Create an area for sterile field and opening packages

What is the correct rationale for using body substance precautions? A) The risk of transmitting HIV in sputum and urine is nonexistent. B) Disease-specific isolation procedures are adequate protection. C) Only actively infected patients are considered contagious. D) All body substances are considered potentially infectious.

D) All body substances are considered potentially infectious.

A nurse working on leadership skills should keep in mind the following accurate statement regarding leaders: A) People are born leaders. B) Leadership should be approached quickly. C) Leaders develop leadership skills in undefined situations. D) All nurse leaders began as inexperienced nurses

D) All nurse leaders began as inexperienced nurses

A nurse is designing a teaching program for individuals who have recently immigrated to the United States from Iraq. Which of the following considerations is necessary for culturally competent patient teaching? A) Use materials developed previously for U.S. citizens. B) Use all visual materials when teaching content. C) Use a lecture format to teach content with few questions. D) Develop written materials in the patients native language.

D) Develop written materials in the patients native language.

A student nurse is performing a urinary catheterization for the first time and inadvertently contaminates the catheter by touching the bed linens. What should the nurse do to maintain surgical asepsis for this procedure? A) Nothing, because the patient is on antibiotics. B) Complete the procedure and then report what happened. C) Apologize to the patient and complete the procedure. D) Gather new sterile supplies and start over.

D) Gather new sterile supplies and start over.

A home health nurse is completing a health history for a patient. What is one question that is important to ask to identify a latex allergy for this patient? A) Have you ever had an allergic reaction to shellfish or iodine? B) Tell me what you use to wash your hands after toileting. C) When you were a child, did you have frequent infections? D) Have you had any unusual symptoms after blowing up balloons?

D) Have you had any unusual symptoms after blowing up balloons?

A 42-year-old male patient recovering from a MI is having difficulty following the care plan to stop smoking and exercise. What is the nurses best response to this patient? A) Praise him for any efforts he makes to improve his health. B) Tell him that he will have another MI and it will be his own fault. C) Tell him that his cigarettes will be taken away if he smokes again. D) Ignore the behavior and recommend a behavior modification program.

D) Ignore the behavior and recommend a behavior modification program.

A nurse is documenting the intensity of a patients pain. What would be the most accurate entry? A) Patient complaining of severe pain. B) Patient appears to be in a lot of pain and is crying. C) Patient states has pain; walking in hall with ease. D) Patient states pain is a 9 on a scale of 1 to 10

D) Patient states pain is a 9 on a scale of 1 to 10

A student is developing a teaching plan for her assigned patient. The student wants to teach the patient about what symptoms to report after chemotherapy. What would the student need to do first? A) Ask other students what should be included in content. B) Ask the patient what he or she wants to know. C) Tell the instructor that this topic hasnt been covered yet. D) Review information available in writing and on the Internet.

D) Review information available in writing and on the Internet.

In Lewins classic theory of change, what happens during unfreezing? A) Planning is conducted. B) Change is initiated. C) Change becomes operational. D) The need for change is recognized.

D) The need for change is recognized.

Which of the following questions asked by the nurse when taking a patients health history would collect data about infection control? A) Tell me what you eat in each 24-hour period. B) Do you sleep well and wake up feeling healthy? C) What were the causes of death for your family members? D) When did you complete your immunizations?

D) When did you complete your immunizations?

A man on an airplane is sitting by a woman who is coughing and sneezing. If she has an infection, what is the most likely means of transmission from the woman to the man? A) direct contact B) indirect contact C) vectors D) airborne route

D) airborne route

What are the recommended cleansing agents for hand hygiene in any setting when the risk of infection is high? A) liquid or bar hand soap B) cold water C) hot water D) antimicrobial products

D) antimicrobial products

Which of the following tasks could be delegated to unlicensed assistive personnel? A) an initial assessment of a patient B) determination of a nursing diagnosis C) evaluation of patient progress with the nursing care plan D) assisting patients with hygiene

D) assisting patients with hygiene

A nurse manager makes all of the decisions for staff activities. What type of leadership is demonstrated by this action? A) democratic B) self-governance C) laissez-faire D) autocratic

D) autocratic

What word or phrase best describes an effective counselor? A) technically skilled B) knowledgeable C) practical D) caring

D) caring

A nurse believes in listening to patients and coworkers more than talking to them, allowing more personal control for all involved. This is a quality of which of the following managerial mind-sets? A) reflective B) analytical C) worldly D) collaborative

D) collaborative

. What term describes a nurse who is sensitive to the patients feelings but remains objective enough to help the patient achieve positive outcomes? A) competent B) caring C) honest D) empathic

D) empathic

What is the most critical element of documentation of teaching? A) a summary of the teaching plan B) the implementation of the teaching plan C) the patient need for learning \ D) evidence that learning has occurred

D) evidence that learning has occurred

A student caring for an unconscious patient knows that communication is important even if the patientdoes not respond. Which nonverbal action by the student would communicate caring? A) making constant eye contact with the patient B) waving to the patient when entering the room C) sighing frequently while providing care D) holding the patients hand while talking

D) holding the patients hand while talking

A specific nursing unit practices functional nursing. What was the basis for this concept? A) individual patient care B) primary nursing C) case management D) industrial assembly line

D) industrial assembly line

. A nurse touches a patients hand to indicate caring and support. What channel of communication is thenurse using? A) auditory B) visual C) olfactory D) kinesthetic

D) kinesthetic

A group of nurses visits selected patients individually at the beginning of each shift. What are these procedures called? A) nursing care conferences B) staff visits C) interdisciplinary referrals D) nursing care rounds

D) nursing care rounds

In which of the following conflict resolution strategies is the conflict rarely resolved? A) collaborating B) compromising C) competing D) smoothing

D) smoothing

A nurse leader is described as charismatic, motivational, and passionate. Communications are open and honest, and the nurse is willing to take risks. What type of leadership is the nurse practicing? A) democratic B) autocratic C) quantum D) transformational

D) transformational

The following procedures have been ordered and implemented for a hospitalized patient. Which procedure carries the greatest risk for a nosocomial infection? A) enema B) intramuscular injections C) heat lamp D) urinary catheterization

D) urinary catheterization

A nurse teaches a rural community group how to avoid contracting West Nile virus by using approved insect repellant and wearing proper coverings when outdoors. By what means is the pathogen involved in West Nile virus transmitted? A) direct contact B) indirect contact C) airborne route D) vectors

D) vectors

. A patient tells the nurse that he is very worried about his surgery. Which of the following responses bythe nurse is a clich? A) Tell me what you are worried about. B) What is it that you are worried about? C) Do you want to cancel your surgery? D) Dont worry, everything will be fine.

D. Dont worry, everything will be fine.

1. A nurse enters the patients room and examines the patients IV fluids and cardiac monitor the patient states well I haven't seen you before who are you? What is the nurse best response? A. Im just the IV therapist checking your IV B. Ive been transferred to this division and will be carrying for you C. Im sorry my name is john smith and I am your nurse D. Im sorry my name is john smith I am your nurse and ill be caring for you until 11pm

D. Im sorry my name is john smith I am your nurse and ill be caring for you until 11pm

1. A nurse enters the room of a patient with cancer the patient is crying and states I feel so alone which response by the nurse is the most therapeutic action A. The nurse stands at the patients bedside and states I understand my mother said the same thing when she was ill B. The nurse places a hand on the patients arm and states you feel so alone C. The nurse stands infront of the patient and ask, why do you feel so alone your wife has been here everyday D. The nurse hold the patients hand and ask what makes you feel so lonely?

D. The nurse hold the patients hand and ask what makes you feel so lonely?

A 3yr old is being admitted to the medical division for vomiting, diarrhea and dehydration. During the admission interview the nurse should implement which communication techniques to elicit the most information from the parents? A. The use of reflective questions B. The use of closed questions C. The use of assertive questions D. The use of clarifying questions

D. the use of clarifying questions

The parents of a school-age child are meeting with the nurse for health promotional education for their child. The child has the following assessment data: a 7-year-old male with diabetes mellitus type 1 with a hemoglobin A1C level of 8.3%, a body mass index (BMI) of 31.7, and a BMI percentile of 99. What are the most appropriate learning diagnoses for this first session? Deficient Knowledge: Readiness for enhanced nutrition, and risk for disturbed body image. Deficient Knowledge: Risk for chronic low self esteem, and risk for unstable blood glucose level. Deficient Knowledge: Risk for imbalanced nutrition: more than body requirements, and sedentary lifestyle. Deficient Knowledge: Imbalanced nutrition: more than body requirements, and ineffective health maintenance.

Deficient Knowledge: Imbalanced nutrition: more than body requirements, and ineffective health maintenance.

A staff nurse works on a medical unit where staff retention is very high. There is a sense of equality between the leader and the staff nurses. The unit decisions and activities are shared between the leader and the group. The designated nurse leader practices which leadership style? Autocratic Democratic Laissez-faire Transformational

Democratic

The nurse has recently been promoted to nurse manager on a pediatric unit and has decided to accept input from staff regarding changes they would like to see on the unit. Which type of leadership style is the nurse embodying? Quantum Autocratic Democratic Laissez-faire

Democratic

The nurse manager who asks staff members to give suggestions on how to improve collaboration between nurses and physicians is exhibiting what style of leadership? Laissez-faire Democratic Autocratic Transactional

Democratic

The nurse is completing documentation after an education session with a client. Which statement best demonstrates detailed documentation of an effective teaching plan? Demonstrated cord care to mother, who stated understanding and performed return demonstration using correct technique. Discussed wet-to-dry dressing changes, and client stated understanding. Spouse taught to flush feeding tube before and after medication. Denied further instruction needed. Lecture provided about infection, and client stated understanding what infection is.

Demonstrated cord care to mother, who stated understanding and performed return demonstration using correct technique

The nurse is completing documentation after an education session with a client. Which statement best demonstrates detailed documentation of an effective teaching plan? Demonstrated cord care to mother, who stated understanding and performed return demonstration using correct technique. Discussed wet-to-dry dressing changes, and client stated understanding. Spouse taught to flush feeding tube before and after medication. Denied further instruction needed. Lecture provided about infection, and client stated understanding what infection is.

Demonstrated cord care to mother, who stated understanding and performed return demonstration using correct technique

A nurse is working with an older adult client, educating the client on how to ambulate with the aid of a walker. The nurse notes that the client appears to lack the motivation to learn how to use the device. The client states, "I'm just too old to learn." What would be most appropriate for the nurse to do to motivate this client? Tell the client how to move the walker as the client ambulates. Explain how the walker supports the client's lower extremities. Fully discuss the rationale for using the walker. Describe how the walker can improve the client's quality of life.

Describe how the walker can improve the client's quality of life

Which of the following is an example of a closed-ended question or statement? A) How did that make you feel? B) Did you take those drugs? C) What medications do you take at home? D) Describe the type of pain you have.

Did you take those drugs?

According to the Canadian Nurses Association (CNA), what is the primary source of evidence to measure performance outcomes against standards of care? Documentation Accreditation Psychomotor skills Clinical judgment

Documentation

1. Which quality in a nurse helps the nurse to become effective in providing for a client's needs while remaining compassionately detached?

Empathy

1. The nurse is visiting a hospice client in the client's home. The client is explaining difficulties with a home infusion pump. By making statements such as "I see" and "go on" during the conversation, the nurse is using which therapeutic nurse-client communication technique?

Encouraging elaboration

The nurse manager is concerned about the large number of teenage mothers being seen in the obstetrics clinic. How can the nurse manager use the transformational leadership style to address the concern? Create a new policy that will limit the number of teenage mothers the clinic can treat. Talk to each teenager who comes in to the clinic about ways to not get pregnant again. Conduct community-based research into the number of teenagers who have become parents over the last five years. Enlist volunteers to help develop a community outreach project that will educate teenagers on methods to prevent pregnancy.

Enlist volunteers to help develop a community outreach project that will educate teenagers on methods to prevent pregnancy

Upon review of a client's microbiology culture results, the nurse recognizes which organism as indicative of normal flora? Escherichia coli in the intestinal tract Escherichia coli in the urinary tract Shigella in the intestinal tract Shigella in the urinary tract

Escherichia coli in the intestinal tract

1. The client is an 18-month-old in the pediatric intensive care unit. The client is scheduled to have a subgaleal shunt placed tomorrow, and the client's mother is quite nervous about the procedure. The nurse tells the client's mother, "The surgeon has done this a million times. Your son will be fine." This is an example of what type of nontherapeutic communication?

False reassurance

An experienced nurse is teaching a student nurse about the proper use of hand hygiene. Which guideline should the nurse provide to the student? The use of gloves eliminates the need for hand hygiene. The use of hand hygiene eliminates the need for gloves. Hand hygiene is needed after contact with objects near the client. Hand lotions should not be used after hand hygiene.

Hand hygiene is needed after contact with objects near the client

A nurse assisting a new mother in the act of breastfeeding represents which form of learning? Affective Psychomotor Cognitive Simplistic

Psychomotor

A nurse is assessing a client who has come to the clinic for a follow-up appointment. The client was diagnosed with asthma several months ago and has missed several appointments since that time. The client also has not been following the medication plan and has not kept the appointment for allergy testing. The nurse suspects that the client may be experiencing problems with health literacy. The nurse teaches the client about the condition and prescribed treatment. Which question from the nurse would help to assess the client's health literacy about the condition? "Do you understand what asthma is?" "How are you supposed to take your medication?" "Are you having trouble getting your medications filled?" "Do you have any questions about what you are supposed to do?"

How are you supposed to take your medication?"

A staff nurse is talking with a clinical nurse leader and asks, "What exactly do you do?" Which statement by the clinical nurse leader would be appropriate? "I'm an administrator involved with client care." "My position is one of management." "I collaborate with health care teams to promote client care." "I'm an advanced practice nurse with a specific specialty area."

I collaborate with health care teams to promote client care."

The nurse is documenting a variance that has occurred during the shift. This report will be used for quality improvement to identify high-risk patterns and, potentially, to initiate in-service programs. This is an example of which type of report? Incident report Nurse's shift report Transfer report Telemedicine report

Incident report

A nurse is preparing to teach a 6-year-old client with a broken arm and the client's mother about caring for the child's cast. Which statement reflects the best education plan for these clients? Include the child in the education; ask questions of both the mother and the child. Focus mainly on the mother; ask the child a couple of simple questions. Provide the mother with written materials; teach the child about keeping the cast dry. Separate the mother and the child; teach the mother and then let the mother teach the child.

Include the child in the education; ask questions of both the mother and the child.

What is the second line of defense in microbial invasion? Inflammation Infection Disease Disability

Inflammation

The health care provider is in a hurry to leave the unit and tells the nurse to give morphine 2 mg IV every 4 hours as needed for pain. What action by the nurse is appropriate? Inform the health care provider that a written order is needed. Write the order in the client's record. Call the pharmacy to have the order entered in the electronic record. Add the new order to the medication administration record.

Inform the health care provider that a written order is needed.

The registered nurse (RN) and unlicensed assistive personnel (UAP) are working together to admit a pediatric client to a nursing unit. Which task would be inappropriate for the RN to delegate to the UAP? Initiating intravenous therapy Securing the client on a papoose board Soothing the client during the procedure Gathering equipment needed for intravenous therapy

Initiating intravenous therapy

Which developmental consideration is a nurse assessing when determining that an 8-year-old child is not equipped to understand the scientific explanation of the child's disease? Intellectual development Motor development Emotional maturity Psychosocial development

Intellectual development

Nurses at a health care facility maintain client records using a method of documentation known as charting by exception (CBE). What is a benefit of this method of documentation? It documents assessments on separate forms. It records progress under problems, intervention, and evaluation. It provides and refers to a client's problem by a number. It provides quick access to abnormal findings.

It provides quick access to abnormal findings

The charge nurse on the orthopedic unit believes in giving the staff as much power as possible. The nurses are allowed, among other things, to create their own work schedules, provide dates and times for unit meetings, and create the agendas, to which the charge nurse contributes. The charge nurse's style of leadership can be described as which? Democratic Laissez-faire Autocratic Transformational

Laissez-faire

The nurse manager calls a staff into a unit meeting to discuss client satisfaction. During the meeting, several staff members assume control. The nurse manager does not intervene to regain control of the group. Which type of leadership style is the nurse embodying? Quantum Autocratic Democratic Laissez-faire

Laissez-faire

The nurse must assign a room for a client admitted with endocarditis and methicillin-resistant Staphylococcus aureus (MRSA) in the blood. A client with which diagnosis can share a room with this client? Vancomycin-resistant enterococci and urinary tract infection Clostridium difficile and colitis Coronary artery bypass grafting MRSA in the wound

MRSA in the wound

The nurse manager for a surgical unit is planning a significant change in how the unit functions. Which action by the nurse manager would be most effective in helping overcome resistance to the planned change? Provide incentives for adopting the change, such as time off. Threaten to end the staff's employment if they do not adopt the change. Demand loyalty and commitment from the staff in adopting the change. Refuse to hear any discussion about adopting the change.

Provide incentives for adopting the change, such as time off.

A nurse is calling a physician to communicate a change in the client's condition. According to the ISBARR format for handoff communication among health care personnel, which is the most appropriate way to begin the conversation?

My name is Sue Smith, RN, and I am calling regarding Mrs. Jones in room 356 at Jefferson Hospital.

A nurse is caring for a client with pneumonia. Which task is most appropriate for the nurse to delegate to an experienced unlicensed assistive personnel (UAP)? Assessing for shortness of breath Instructing the client about the need to alternate activity with rest Obtaining vital signs every 4 hours Administering nebulizer treatments as needed

Obtaining vital signs every 4 hours

Which abbreviation is correct for use in documentation? PO Sub q Per os BT

PO

The nurse prepares for a sterile procedure. Of those listed, what action does the nurse perform first? Put on personal protective equipment, if required. Perform hand hygiene. Check that the packaged kit is dry and unopened. Set up a work area at waist level.

Perform hand hygiene

An active, otherwise healthy, older adult client presents to the clinic with severe osteoarthritis in both knees. The nurse knows this client does not want to be a burden on the family, and the client remains stoic despite reporting the pain as severe. The client avoids the topic of surgery and attends church weekly. The client's family is supportive of any decisions the client makes regarding health. Which of the assessment data is most important to forming an individualized education plan for this client concerning treatment for osteoarthritis? Orthopedic surgical history Personal perception of health and aging Floor plan of the client's dwelling Formal religious beliefs

Personal perception of health and aging

A 46-year-old obese client has been diagnosed with hypertension and type 2 diabetes. The client acknowledges the need to lose weight. The client recently visited a local fitness club, obtained a membership, and has signed up for their next water aerobics class. According to the Transtheoretical Model of Change, what stage of change is this client in related to her weight loss? Preparation Maintenance Precontemplation Contemplation

Preparation

The nurses at a health care facility were informed of the change to organize the clients' records into problem-oriented records. Which explanation could assist the nurses in determining the advantage of using problem-oriented records? Problem-oriented recording gives clients the right to withhold the release of their information to anyone. Problem-oriented recording makes it difficult to demonstrate a unified approach for resolving clients' problems among caregivers. Problem-oriented recording emphasizes goal-directed care to promote the recording of pertinent data that will facilitate communication among health care providers. Problem-oriented recording has numerous locations for information where members of the multidisciplinary team can make entries about their own specific activities in relation to the client's care.

Problem-oriented recording emphasizes goal-directed care to promote the recording of pertinent data that will facilitate communication among health care providers.

When caring for a diabetic client, the nurse notes that the client learns better when practicing the self-administration of the insulin injection alone. In which learning domain does this client's learning style fall? Cognitive Affective Psychomotor Interpersonal

Psychomotor

Nurses working in bed management are assigning clients from the emergency room to semiprivate rooms. Clients with which two diagnoses are appropriate to room together, based on safety and infection control standards? Clostridium difficile and diabetic ketoacidosis Reactive airway disease and exacerbation of chronic obstructive pulmonary disorder (COPD) Tuberculosis and pneumonia Appendectomy and a draining leg ulcer positive for methicillin-resistant Staphylococcus aureus

Reactive airway disease and exacerbation of chronic obstructive pulmonary disorder (COPD

Nurses working in bed management are assigning clients from the emergency room to semiprivate rooms. Clients with which two diagnoses are appropriate to room together, based on safety and infection control standards? Clostridium difficile and diabetic ketoacidosis Reactive airway disease and exacerbation of chronic obstructive pulmonary disorder (COPD) Tuberculosis and pneumonia Appendectomy and a draining leg ulcer positive for methicillin-resistant Staphylococcus aureus

Reactive airway disease and exacerbation of chronic obstructive pulmonary disorder (COPD)

What is an accurate guideline for the use of PPE? Put on PPE after entering the client's room. Substitute personal glasses for protective eyewear, if desired. Replace gloves if they are visibly soiled. When wearing gloves, work from "dirty" areas to "clean" ones.

Replace gloves if they are visibly soiled

The nurse is sharing information about a client at change of shift. The nurse is performing what nursing action? Dialogue Documentation Reporting Verification

Report

The nurse is caring for a client who requests to see a copy of the client's own health care records. What action by the nurse is most appropriate? Review the hospital's process for allowing clients to view their health care records. Access the health care record at the bedside and show the client how to navigate the electronic health record. Discuss how the hospital can be fined for allowing clients to view their health care records. Explain that only a paper copy of the health care record can be viewed by the client.

Review the hospital's process for allowing clients to view their health care records.

The nurse is caring for a client who has an elevated temperature. When calling the health care provider, the nurse should use which communication tools to ensure that communication is clear and concise? SBAR SOAP PIE MAR

SBAR

When recording data regarding the client's health record, the nurse mentions the analysis of the subjective and objective data, in addition to detailing the plan for care of the client. Which of the following styles of documentation is the nurse implementing? FOCUS charting SOAP charting PIE charting narrative charting

SOAP charting

A nurse may attempt to help a client solve a situational crisis during what type of counseling session? Long-term counseling Motivational counseling Short-term counseling Professional counseling

Short-term counseling

Which of these statements reflects the expected functioning at a hospital that has achieved Magnet status? Nursing administration is in control of all decision-making. Most client outcomes have improved but are not at target range. Staff nurses are developing innovative solutions to problems . There is a decreased rate of retention among the nursing staff.

Staff nurses are developing innovative solutions to problems

1. A client is reluctant to undergo surgery and is discussing it with the nurse. Which response by the nurse would reflect an authoritarian approach?

Surgery is your only option. You need this operation

A client is reluctant to learn to do finger sticks for home international normalized ratio (INR) monitoring. What is the best statement by the nurse? Are you worried about the pain? Why don't you want to do this? Most people are afraid of sticking themselves. Tell me what you know about these tests.

Tell me what you know about these tests

The student nurse observes another nurse wash her hands in the client's bathroom before exiting the room. This client's stool came back positive (+) for Clostridium Difficile (C diff). Why is this behavior incorrect? Clostridium difficile bacteria is eradicated by the use of hand sanitizer only. The behavior is not a problem as long as the nurse uses gloves in the room. The nurse must make sure that the bathroom has been cleaned recently before washing her hands. The bathroom is highly contaminated with the Clostridium difficile bacteria.

The bathroom is highly contaminated with the Clostridium difficile bacteria.

A charge nurse on a medical-surgical unit is asked by the nurse manager to serve as a mentor to another staff nurse who is less experienced. Which of these would best describe this role? The charge nurse is being paid to supervise the staff nurse. The staff nurse is learning about all the hospital policies from the charge nurse. The charge nurse is providing support for the staff nurse in new responsibilities. The staff nurse is orienting to the unit as a newly hired nurse.

The charge nurse is providing support for the staff nurse in new responsibilities.

When establishing a teaching-learning relationship with a client, it is most important for the nurse to remember that effective learning can best be achieved through which concept? The nurse is the expert in the teaching-learning environment. The nurse must be able to handle criticism during the process. The client and the nurse are equal participants. Assimilation and application of psychomotor concepts is essential.

The client and the nurse are equal participants.

A nurse is educating a client with a new diagnosis of diabetes. Which example demonstrates cognitive learning by the client? The client describes signs and symptoms of hypoglycemia. The client demonstrates proper technique for injecting insulin. The client expresses a desire to improve nutritional intake and lose weight. The client prepares the skin for the administration of an insulin injection.

The client describes signs and symptoms of hypoglycemia.

A client, eager to go home from an acute care facility, calls out to have discharge education completed. The nurse is not able to get to the client's room until an hour later, and finds the client asleep. The client's significant other states, "She will be out for a couple hours after that pain medication." Which of the following best describes what must happen with the education session? The nurse cannot determine the subject matter to teach if the client is asleep. The client is not demonstrating readiness to learn due to the effects of medication. The significant other can be taught now, and then teach the client later on The client can be awakened when she falls asleep during the session.

The client is not demonstrating readiness to learn due to the effects of medication.

The nurse is educating a client regarding a new skill. When evaluating the client's knowledge about the topic covered, which best represents that the client has learned a new skill? The client states understanding and passes a written test. The client organizes materials needed and gives return demonstration. The client verbalizes items needed and how to perform the skill. The client nods when asked about process and assists with cleanup.

The client organizes materials needed and gives return demonstration

The nurse is reassessing a client after pain medication has been administered to manage the pain from a bilateral knee replacement procedure. Which statement most accurately depicts proper documentation of pain assessment? The client is receiving sufficient relief from pain medication, stating no pain in either knee. The client appears comfortable and is resting adequately and appears to not be in acute distress. The client reports that on a scale of 0 to 10, the current pain is a 3. The client appears to have a low tolerance for pain and frequently reports intense pain.

The client reports that on a scale of 0 to 10, the current pain is a 3.

The nurse is providing instructions to a client about performance of breast self-examination. What learning outcome would be most appropriate regarding this education? The client will demonstrate self-efficacy and improved body image. The client will have restoration of breast function. The client will be able to perform proper breast self-examination for breast cancer detection and prevention. The client will demonstrate improved coping skills.

The client will be able to perform proper breast self-examination for breast cancer detection and prevention

A client has a nursing diagnosis of Deficient Knowledge related to prescribed antibiotic therapy. Which outcome would the nurse identify as most appropriate? The client will state how to safely take the prescribed antibiotic. The client will identify signs and symptoms of worsening infection. The client will verbalize measures appropriate to minimize infection transmission. The client demonstrates the proper technique for hand hygiene.

The client will state how to safely take the prescribed antibiotic.

A client has a diagnosis of HIV and has been admitted to the hospital with an opportunistic infection that originated with the client's normal flora. Why did this client most likely become ill from his resident microorganisms? The resident microorganisms mutated and became virulent The client's immune system became further weakened The client's normal flora proliferated because of a nutritional deficit The client's normal flora began producing spores

The client's immune system became further weakened

Which example may illustrate a breach of confidentiality and security of client information? The nurse provides information over the phone to the client's family member who lives in a neighboring state. The nurse provides information to a professional caregiver involved in the care of the client. The nurse informs a colleague that she should not be discussing client information in the hospital cafeteria. The nurse accesses client information on the computer at the nurses' station, then logs off before answering a client's call bell.

The nurse provides information over the phone to the client's family member who lives in a neighboring state

A nurse is writing learner objectives for a client who was recently diagnosed with type 2 diabetes. Which statement best describes the proper method for writing objectives? The nurse writes one or two broad objectives rather than several specific objectives. The nurse writes general statements for learner objectives that could be accomplished in any amount of time. The nurse plans learner objectives with another nurse before obtaining input from the client and family. The nurse writes one long-term objective for each diagnosis, followed by several specific objectives.

The nurse writes one long-term objective for each diagnosis, followed by several specific objectives.

a parish nurse is preparing to provide a health promotion class to a group of adults in the parish. In preparing to meet the learning needs of this group, the nurse recognizes which as a characteristic of an adult learner? Their readiness to learn is often related to a developmental task or social role. Peer group acceptance is a critical issue for this age group. The material presented should focus on future application. Previous experiences have little impact on learning.

Their readiness to learn is often related to a developmental task or social role

A parish nurse is preparing to provide a health promotion class to a group of adults in the parish. In preparing to meet the learning needs of this group, the nurse recognizes which as a characteristic of an adult learner? Their readiness to learn is often related to a developmental task or social role. Peer group acceptance is a critical issue for this age group. The material presented should focus on future application. Previous experiences have little impact on learning.

Their readiness to learn is often related to a developmental task or social role.

The client presents to the Emergency Department reporting fever, chills, and a productive cough. The chest x-ray shows an area of infiltrate, and the primary care provider prescribes a broad spectrum antibiotic. Which client teaching statement from the nurse is most appropriate regarding the prescribed drug? "This antibiotic is the best choice since the causative organism is not known." "This antibiotic causes fewer side effects than a narrow spectrum antibiotic." "Drug resistance can develop when the wrong antibiotic is used for pneumonia." "Pneumonia is usually caused by multiple organisms."

This antibiotic is the best choice since the causative organism is not known.

A client has requested a translator to help understand the questions that the nurse is asking during the client interview. The nurse knows that what is important when working with a client translator? Talking directly to the translator facilitates the transfer of information. Talking loudly helps the translator and the client understand the information better. It is always okay to not use a translator if a family member can do it. Translators may need additional explanations of medical terms.

Translators may need additional explanations of medical terms

A client has requested a translator to help understand the questions that the nurse is asking during the client interview. The nurse knows that what is important when working with a client translator? Talking directly to the translator facilitates the transfer of information. Talking loudly helps the translator and the client understand the information better. It is always okay to not use a translator if a family member can do it. Translators may need additional explanations of medical terms.

Translators may need additional explanations of medical terms

Following a myocardial infarction (heart attack), a client begins to recognize the need to increase exercise, eat a low-fat diet, and implement relaxation techniques. According to Lewin, this client is in which stage of the change process? Prizing Unfreezing Moving Refreezing

Unfreezing

While the nurse is caring for a hearing impaired client, and a family member of the client states, "What do you think is the best way to communicate?" What is the best response by the nurse? "Use words that begin with 'f,' 's,' 'k,' and 'sh' to communicate." "Use flash cards and writing pads." "Limit communication to avoid frustration." "Encourage family members to increase their vocal pitch."

Use flash cards and writing pads.

A nurse is trying to encourage a client with paraplegia who is depressed and not adhering to the treatment program to join a support group. Which statement by the nurse is most appropriate? "What do you know about support groups?" "Support groups are for people like you who are depressed." "I am sure you would feel better if you joined a support group." "I am going to sign you up for a support group."

What do you know about support groups?"

during rounds a charge nurse hears the patient care technician yelling loudly to a patient regarding a transfer from the bed to chair. Upon entering the room what is the nurse best response?

When your patient is safe and comfortable, meet me at the desk

A community health nurse provides information to a client with newly diagnosed multiple sclerosis about a support group at the local hospital for clients with the disease and their families. Providing this information is an example of: a referral. a consultation. conferring. reporting.

a referral

The nurse caring for clients at an outpatient clinic determines that which client is at greatest risk for infection? an 80-year-old woman a 2-year-old toddler a 12-year-old girl an 18-month-old infant

an 80-year-old woman

For which client would the use of standard precautions alone be appropriate? a client with diphtheria who needs p.m. care a client with TB who needs medications administered an incontinent client in a nursing home who has diarrhea a child with chickenpox who is treated in the emergency room

an incontinent client in a nursing home who has diarrhea

The nurse is reviewing the plan of care for assigned clients. Which client has the highest risk for developing an infection? an older adult client with a history of heart failure a school-age child who is current with immunizations an adolescent who has a right radial fracture a middle-aged adult who takes prescribed medication to control blood pressure

an older adult client with a history of heart failure

A nurse during orientation notices that the preceptor gives all subcutaneous injections on a 45-degree angle. When the new nurse asks the preceptor the rationale for the practice the preceptors states, "This is how I do it, and this is how you will do it." The new nurse recognizes this behavior to be:

assertive

The nurse is caring for a client with a surgical wound. Which action by the nurse best reduces the reservoir of infection? changing the soiled dressing wearing clean unsterile gloves when changing the dressing isolating the client's belongings applying a face mask with shield

changing the soiled dressing

The nurse is caring for a client with hypertension, and only documents a blood pressure of 170/100 mmHg when all other vital signs are normal. This reflects what type of documentation? SOAP narrative focus charting by exception

charting by exception

A nurse suspects that a client has a respiratory infection. Which symptom would the nurse be least likely to assess? clear mucus productive cough dyspnea abnormal breath sounds

clear mucus

The nurse is admitting a client who has a draining wound that is contaminated with Staphylococcus aureus. What type of precautions should the nurse initiate for this client? droplet precautions airborne precautions neutropenic precautions contact precautions

contact

The nurse is caring for a 27-year-old client who presents with possible signs of an infected abdominal wound. Which action should the nurse prioritize and initiate after receiving the results of the laboratory test indicating the client has methicillin-resistant Staphylococcus aureus (MRSA) infection? airborne droplet contact reverse isolation

contact

After educating students about changes in the immune system and risk for infection as people age, the instructor determines that the education was successful when the students identify: increased humoral immunity response. decreased cellular immunity. increased effectiveness of phagocytosis. decreased susceptibility to infection.

decreased cellular immunity.

The process of phagocytosis involves: secretion of a nonspecific chemical inhibitor. depletion of serotonin in the brain cells. digestion of microbes by white blood cells. breakdown of proteins into amino acids.

digestion of microbes by white blood cells.

1. Touch is a personal behavior that means the same thing to all persons.

false

According to the Health Insurance Portability and Accountability Act (HIPAA) passed in 1996, clients: have the right to copy their health records. need to obtain legal representation to update their health records. can be punished for violating guidelines. are required to obtain health record information through their insurance company.

have the right to copy their health records.

A physician performs lumbar puncture and advises the nurse to send the obtained cerebrospinal fluid for Gram stains. The nurse understands that this type of testing is beneficial for which reason? permits selection of antibiotic concentration helps in reducing proliferation of multidrug-resistant organisms narrows the therapeutic range to avoid prolonged use helps to determine prescribed antibiotic therapy

helps to determine prescribed antibiotic therapy

A nurse on a night shift entered an older adult client's room during a scheduled check and discovered the client on the floor beside the bed, the result of falling when trying to ambulate to the washroom. After assessing the client and assisting into the bed, the nurse has completed an incident report. What is the primary purpose of this particular type of documentation? identifying risks and ensuring future safety for clients gauging the nurse's professional performance over time protecting the nurse and the hospital from litigation following up the incident with other members of the care team

identifying risks and ensuring future safety for clients

To eliminate needlesticks as potential hazards to nurses, the nurse should: place the uncapped needle on a tray and carry it to the medicine room for disposal. immediately deposit uncapped needles into a puncture-proof plastic container. stick the uncapped needle into a Styrofoam block and deposit it in a plastic container. slide the needle into the cap and deposit it in a puncture-proof plastic container.

immediately deposit uncapped needles into a puncture-proof plastic container

The nurse is receiving a confused client with a draining wound onto the medical-surgical unit. Which room assignment will the nurse make? into a private room with a client with pneumonia with a client with a myocardial infarction with another client with a draining wound

into a private room

What action by the nurse will facilitate the helping relationship during the orientation phase? A) providing assistance to meet activities of daily living B) introducing himself or herself to the patient by name C) designing a specific teaching plan of care D) preparing for termination of the relationship

introducing himself or herself to the patient by name

When documenting the care of a client, the nurse is aware of the need to use abbreviations conscientiously and safely. This includes: limiting abbreviations to those approved for use by the institution. using only abbreviations whose meaning is self-evident to an educated health professional. ensuring that abbreviations are understandable to clients who may seek access to their health records. using only those abbreviations that are defined in full at another location in the client's chart.

limiting abbreviations to those approved for use by the institution.

A client comes to the emergency department reporting becoming very ill after consuming shrimp and lobster. How will the nurse document this condition? contagious disease infectious disease communicable disease noncommunicable disease

noncommunicable disease

The nurse is caring for a client who demonstrates a health literacy concern. The nurse adjusts client teaching in which way? uses medical terminology to help the client feel smarter provides general teaching instead of specificity regarding diagnosis gives instructions in multiple ways so the client will understand uses videos, diagrams, and pictures rather than focusing on verbal teaching

uses videos, diagrams, and pictures rather than focusing on verbal teaching

nfection occurs when the host is exposed to pathogens. What type of pathogen uses the cell's metabolism, and replicates itself while destroying the cell or changing the cell's genetic makeup? Bacteria Fungi Virus Parasites

virus

Which care intervention should the nurse anticipate when providing care to a client admitted with a possible diagnosis of tuberculosis (TB)? wearing a particulate respirator for all care and interaction with this client wearing a face mask when entering and staying at a distance from the client wearing protective eye wear for contact with this client placing the client in a regular, private room

wearing a particulate respirator for all care and interaction with this client


Related study sets

Ch21a-Explain the accounting for operating leases

View Set

(PrepU) Chapter 16: Postoperative Nursing Management

View Set

Circles in the Coordinate Plane Quiz 2023-2024

View Set

ServSafe Chapter 6: The Flow of Food: Purchasing and Receiving

View Set